Is $(sum_{k=0}^n sin(k!))_{n in mathbb{N}} $ bounded?












5












$begingroup$


It is a relatively easy exercice to show that $(sum_{k=0}^n sin(k))_{n in mathbb{N}} $ is bounded by considering complex exponentials $e^{ik}$ (that form a geometric sum) and then taking the imaginary part. But what happens when we replace $k$ by $k!$ ?



I think that it is bounded by analogy. The terms somehow compensate one another... But I was unsucessfull at proving it.



One of my attempts : I tried to prove that $n!$ is an equidistributed sequence mod $2 pi$ (I'm not sure that's true), but even with that I struggle to conclude.










share|cite|improve this question











$endgroup$












  • $begingroup$
    Well, $sum_{k=0}^nsin(n!pi e)$ is bounded, due to the Taylor expansion of $e^x$, but I doubt that'll help you here.
    $endgroup$
    – Simply Beautiful Art
    Jan 9 at 20:45






  • 4




    $begingroup$
    It should be noted that, for instance, the sequence $sum_{k=0}^nsin(k^2)$ are not bounded, even though squares are equidistributed modulo $2pi$. Your problem is probably even harder than that, since I doubt we know much about distribution of $k!$ mod $2pi$.
    $endgroup$
    – Wojowu
    Jan 9 at 20:46








  • 2




    $begingroup$
    Reference to my previous comment
    $endgroup$
    – Wojowu
    Jan 9 at 20:47






  • 2




    $begingroup$
    @Wojowu Why "even though" they are equidistributed? If $X_n$ were i.i.d. random variables uniformly distributed on $S^1$, I would expect $sum_{k=0}^n sin (X_k)$ to be almost surely unbounded since the standard deviation would be proportional to $sqrt{n+1}$.
    $endgroup$
    – Daniel Schepler
    Jan 9 at 20:54






  • 1




    $begingroup$
    @DanielSchepler "even though" because this has been OP's attempt (and that example implies this approach can't work).
    $endgroup$
    – Wojowu
    Jan 9 at 20:56


















5












$begingroup$


It is a relatively easy exercice to show that $(sum_{k=0}^n sin(k))_{n in mathbb{N}} $ is bounded by considering complex exponentials $e^{ik}$ (that form a geometric sum) and then taking the imaginary part. But what happens when we replace $k$ by $k!$ ?



I think that it is bounded by analogy. The terms somehow compensate one another... But I was unsucessfull at proving it.



One of my attempts : I tried to prove that $n!$ is an equidistributed sequence mod $2 pi$ (I'm not sure that's true), but even with that I struggle to conclude.










share|cite|improve this question











$endgroup$












  • $begingroup$
    Well, $sum_{k=0}^nsin(n!pi e)$ is bounded, due to the Taylor expansion of $e^x$, but I doubt that'll help you here.
    $endgroup$
    – Simply Beautiful Art
    Jan 9 at 20:45






  • 4




    $begingroup$
    It should be noted that, for instance, the sequence $sum_{k=0}^nsin(k^2)$ are not bounded, even though squares are equidistributed modulo $2pi$. Your problem is probably even harder than that, since I doubt we know much about distribution of $k!$ mod $2pi$.
    $endgroup$
    – Wojowu
    Jan 9 at 20:46








  • 2




    $begingroup$
    Reference to my previous comment
    $endgroup$
    – Wojowu
    Jan 9 at 20:47






  • 2




    $begingroup$
    @Wojowu Why "even though" they are equidistributed? If $X_n$ were i.i.d. random variables uniformly distributed on $S^1$, I would expect $sum_{k=0}^n sin (X_k)$ to be almost surely unbounded since the standard deviation would be proportional to $sqrt{n+1}$.
    $endgroup$
    – Daniel Schepler
    Jan 9 at 20:54






  • 1




    $begingroup$
    @DanielSchepler "even though" because this has been OP's attempt (and that example implies this approach can't work).
    $endgroup$
    – Wojowu
    Jan 9 at 20:56
















5












5








5


3



$begingroup$


It is a relatively easy exercice to show that $(sum_{k=0}^n sin(k))_{n in mathbb{N}} $ is bounded by considering complex exponentials $e^{ik}$ (that form a geometric sum) and then taking the imaginary part. But what happens when we replace $k$ by $k!$ ?



I think that it is bounded by analogy. The terms somehow compensate one another... But I was unsucessfull at proving it.



One of my attempts : I tried to prove that $n!$ is an equidistributed sequence mod $2 pi$ (I'm not sure that's true), but even with that I struggle to conclude.










share|cite|improve this question











$endgroup$




It is a relatively easy exercice to show that $(sum_{k=0}^n sin(k))_{n in mathbb{N}} $ is bounded by considering complex exponentials $e^{ik}$ (that form a geometric sum) and then taking the imaginary part. But what happens when we replace $k$ by $k!$ ?



I think that it is bounded by analogy. The terms somehow compensate one another... But I was unsucessfull at proving it.



One of my attempts : I tried to prove that $n!$ is an equidistributed sequence mod $2 pi$ (I'm not sure that's true), but even with that I struggle to conclude.







real-analysis sequences-and-series






share|cite|improve this question















share|cite|improve this question













share|cite|improve this question




share|cite|improve this question








edited Jan 9 at 20:36







C. Lauverjat

















asked Jan 9 at 20:24









C. LauverjatC. Lauverjat

262




262












  • $begingroup$
    Well, $sum_{k=0}^nsin(n!pi e)$ is bounded, due to the Taylor expansion of $e^x$, but I doubt that'll help you here.
    $endgroup$
    – Simply Beautiful Art
    Jan 9 at 20:45






  • 4




    $begingroup$
    It should be noted that, for instance, the sequence $sum_{k=0}^nsin(k^2)$ are not bounded, even though squares are equidistributed modulo $2pi$. Your problem is probably even harder than that, since I doubt we know much about distribution of $k!$ mod $2pi$.
    $endgroup$
    – Wojowu
    Jan 9 at 20:46








  • 2




    $begingroup$
    Reference to my previous comment
    $endgroup$
    – Wojowu
    Jan 9 at 20:47






  • 2




    $begingroup$
    @Wojowu Why "even though" they are equidistributed? If $X_n$ were i.i.d. random variables uniformly distributed on $S^1$, I would expect $sum_{k=0}^n sin (X_k)$ to be almost surely unbounded since the standard deviation would be proportional to $sqrt{n+1}$.
    $endgroup$
    – Daniel Schepler
    Jan 9 at 20:54






  • 1




    $begingroup$
    @DanielSchepler "even though" because this has been OP's attempt (and that example implies this approach can't work).
    $endgroup$
    – Wojowu
    Jan 9 at 20:56




















  • $begingroup$
    Well, $sum_{k=0}^nsin(n!pi e)$ is bounded, due to the Taylor expansion of $e^x$, but I doubt that'll help you here.
    $endgroup$
    – Simply Beautiful Art
    Jan 9 at 20:45






  • 4




    $begingroup$
    It should be noted that, for instance, the sequence $sum_{k=0}^nsin(k^2)$ are not bounded, even though squares are equidistributed modulo $2pi$. Your problem is probably even harder than that, since I doubt we know much about distribution of $k!$ mod $2pi$.
    $endgroup$
    – Wojowu
    Jan 9 at 20:46








  • 2




    $begingroup$
    Reference to my previous comment
    $endgroup$
    – Wojowu
    Jan 9 at 20:47






  • 2




    $begingroup$
    @Wojowu Why "even though" they are equidistributed? If $X_n$ were i.i.d. random variables uniformly distributed on $S^1$, I would expect $sum_{k=0}^n sin (X_k)$ to be almost surely unbounded since the standard deviation would be proportional to $sqrt{n+1}$.
    $endgroup$
    – Daniel Schepler
    Jan 9 at 20:54






  • 1




    $begingroup$
    @DanielSchepler "even though" because this has been OP's attempt (and that example implies this approach can't work).
    $endgroup$
    – Wojowu
    Jan 9 at 20:56


















$begingroup$
Well, $sum_{k=0}^nsin(n!pi e)$ is bounded, due to the Taylor expansion of $e^x$, but I doubt that'll help you here.
$endgroup$
– Simply Beautiful Art
Jan 9 at 20:45




$begingroup$
Well, $sum_{k=0}^nsin(n!pi e)$ is bounded, due to the Taylor expansion of $e^x$, but I doubt that'll help you here.
$endgroup$
– Simply Beautiful Art
Jan 9 at 20:45




4




4




$begingroup$
It should be noted that, for instance, the sequence $sum_{k=0}^nsin(k^2)$ are not bounded, even though squares are equidistributed modulo $2pi$. Your problem is probably even harder than that, since I doubt we know much about distribution of $k!$ mod $2pi$.
$endgroup$
– Wojowu
Jan 9 at 20:46






$begingroup$
It should be noted that, for instance, the sequence $sum_{k=0}^nsin(k^2)$ are not bounded, even though squares are equidistributed modulo $2pi$. Your problem is probably even harder than that, since I doubt we know much about distribution of $k!$ mod $2pi$.
$endgroup$
– Wojowu
Jan 9 at 20:46






2




2




$begingroup$
Reference to my previous comment
$endgroup$
– Wojowu
Jan 9 at 20:47




$begingroup$
Reference to my previous comment
$endgroup$
– Wojowu
Jan 9 at 20:47




2




2




$begingroup$
@Wojowu Why "even though" they are equidistributed? If $X_n$ were i.i.d. random variables uniformly distributed on $S^1$, I would expect $sum_{k=0}^n sin (X_k)$ to be almost surely unbounded since the standard deviation would be proportional to $sqrt{n+1}$.
$endgroup$
– Daniel Schepler
Jan 9 at 20:54




$begingroup$
@Wojowu Why "even though" they are equidistributed? If $X_n$ were i.i.d. random variables uniformly distributed on $S^1$, I would expect $sum_{k=0}^n sin (X_k)$ to be almost surely unbounded since the standard deviation would be proportional to $sqrt{n+1}$.
$endgroup$
– Daniel Schepler
Jan 9 at 20:54




1




1




$begingroup$
@DanielSchepler "even though" because this has been OP's attempt (and that example implies this approach can't work).
$endgroup$
– Wojowu
Jan 9 at 20:56






$begingroup$
@DanielSchepler "even though" because this has been OP's attempt (and that example implies this approach can't work).
$endgroup$
– Wojowu
Jan 9 at 20:56












0






active

oldest

votes











Your Answer





StackExchange.ifUsing("editor", function () {
return StackExchange.using("mathjaxEditing", function () {
StackExchange.MarkdownEditor.creationCallbacks.add(function (editor, postfix) {
StackExchange.mathjaxEditing.prepareWmdForMathJax(editor, postfix, [["$", "$"], ["\\(","\\)"]]);
});
});
}, "mathjax-editing");

StackExchange.ready(function() {
var channelOptions = {
tags: "".split(" "),
id: "69"
};
initTagRenderer("".split(" "), "".split(" "), channelOptions);

StackExchange.using("externalEditor", function() {
// Have to fire editor after snippets, if snippets enabled
if (StackExchange.settings.snippets.snippetsEnabled) {
StackExchange.using("snippets", function() {
createEditor();
});
}
else {
createEditor();
}
});

function createEditor() {
StackExchange.prepareEditor({
heartbeatType: 'answer',
autoActivateHeartbeat: false,
convertImagesToLinks: true,
noModals: true,
showLowRepImageUploadWarning: true,
reputationToPostImages: 10,
bindNavPrevention: true,
postfix: "",
imageUploader: {
brandingHtml: "Powered by u003ca class="icon-imgur-white" href="https://imgur.com/"u003eu003c/au003e",
contentPolicyHtml: "User contributions licensed under u003ca href="https://creativecommons.org/licenses/by-sa/3.0/"u003ecc by-sa 3.0 with attribution requiredu003c/au003e u003ca href="https://stackoverflow.com/legal/content-policy"u003e(content policy)u003c/au003e",
allowUrls: true
},
noCode: true, onDemand: true,
discardSelector: ".discard-answer"
,immediatelyShowMarkdownHelp:true
});


}
});














draft saved

draft discarded


















StackExchange.ready(
function () {
StackExchange.openid.initPostLogin('.new-post-login', 'https%3a%2f%2fmath.stackexchange.com%2fquestions%2f3067909%2fis-sum-k-0n-sink-n-in-mathbbn-bounded%23new-answer', 'question_page');
}
);

Post as a guest















Required, but never shown

























0






active

oldest

votes








0






active

oldest

votes









active

oldest

votes






active

oldest

votes
















draft saved

draft discarded




















































Thanks for contributing an answer to Mathematics Stack Exchange!


  • Please be sure to answer the question. Provide details and share your research!

But avoid



  • Asking for help, clarification, or responding to other answers.

  • Making statements based on opinion; back them up with references or personal experience.


Use MathJax to format equations. MathJax reference.


To learn more, see our tips on writing great answers.




draft saved


draft discarded














StackExchange.ready(
function () {
StackExchange.openid.initPostLogin('.new-post-login', 'https%3a%2f%2fmath.stackexchange.com%2fquestions%2f3067909%2fis-sum-k-0n-sink-n-in-mathbbn-bounded%23new-answer', 'question_page');
}
);

Post as a guest















Required, but never shown





















































Required, but never shown














Required, but never shown












Required, but never shown







Required, but never shown

































Required, but never shown














Required, but never shown












Required, but never shown







Required, but never shown







Popular posts from this blog

Mario Kart Wii

What does “Dominus providebit” mean?

Antonio Litta Visconti Arese